¿Cómo predice QFT la densidad de probabilidad de encontrar una partícula en x?

En mecánica cuántica, la densidad de probabilidad de la posición de una partícula es

ρ ( X ) = | X | ψ | 2

¿Cuál es la expresión correspondiente en QFT para predecir esta distribución? Ya que ρ ( X ) se puede medir (al menos con cierta precisión) en un experimento, esta parece ser una pregunta justa.

He hecho esta pregunta a algunas personas en mi universidad. Para anticipar algunas respuestas que, en mi opinión, no resolvieron esta pregunta, permítanme señalar que

  • En QFT, debido a que el número de partículas no se conserva, se ha sugerido que la pregunta que hago podría no tener sentido. Sin embargo, una versión ligeramente reformulada de la pregunta sí lo hace: "¿cuál es la densidad de probabilidad de encontrar (por ejemplo) exactamente un electrón en x?".
  • He leído algunas respuestas a preguntas similares como esta , que menciona un operador de posición definido por Newton & Wigner, de modo que sus estados propios dan la probabilidad de un resultado de posición como en QM. Según la misma publicación, este intento de un operador X falló porque la probabilidad no era invariante de Lorentz.
  • U(1) simetría en L sugiere algo así como una densidad de probabilidad, pero esta cantidad puede ser negativa, por lo que se interpreta como densidad de carga (hasta donde he leído).
  • Soy consciente de que QFT generalmente no se usa para predecir densidades de probabilidad de posición. Sin embargo, tales cosas se miden comúnmente, por ejemplo, en el experimento de la doble rendija. Por lo tanto, si la teoría es al menos tan general como QM, es deseable que tenga algo que decir aquí.

Por supuesto, si uno de estos puntos de alguna manera resuelve la pregunta, me interesaría mucho entender mejor cómo.

Es posible que desee echar un vistazo a mi pregunta similar aquí: physics.stackexchange.com/q/287759/110516 .
El supuesto explícito de QFT de interacción perturbativa en el espacio-tiempo de Minkowski es que las partículas son libres (es decir, descritas por campos cuánticos libres) en tiempos asintóticos, por lo tanto, solo las amplitudes de probabilidad de dispersión son observables de la teoría. Quizás @Arnold Neumaier pueda decirnos qué sucede en el QFT no perturbativo (también llamado QFT constructivo).
@DanielC Para mi mente ingenua, parece correcto tomar el límite de dispersión y aún esperar predicciones para las mediciones de posición. Por ejemplo, en el experimento de la doble rendija, la pantalla puede estar tan lejos de las rendijas como (digamos) la distancia entre el centro de dispersión y el detector en el LHC.
Posiblemente relacionado . Pero tenga en cuenta, como en esa respuesta, que QFT se usa en experimentos de dispersión, esencialmente en el espacio de impulso: uno profundiza indirectamente en distancias cortas. La hiperlocalización está esencialmente fuera de lugar. QFT es difícilmente aplicable a experimentos de hendidura y similares.

Respuestas (1)

La amplitud requerida es una superposición entre un estado de partícula localizado y un estado de configuración de campo.

Para el caso de un campo escalar libre, Roman Jackiw en su trabajo sobre Representaciones funcionales para campos cuantificados ha realizado el cálculo de la amplitud exactamente requerida (ecuación 2.14A).

Repetiré aquí algunos detalles de su derivación y reformularé el resultado en una base de estado coherente, que creo que es más susceptible de medición experimental.

En la imagen de Schrödinger, asociamos un vector | ϕ en el espacio de Hilbert a cada configuración de campo. Tenga en cuenta que todas las siguientes expresiones son generalizaciones de dimensión infinita del oscilador armónico. (Los siguientes cálculos no son rigurosos, el rigor puede mejorarse ponderando y difuminando, pero esto no se hará aquí).

La acción del operador de campo en el momento t = 0 sobre los vectores de configuración viene dada por:

Φ ^ ( X ) | ϕ = ϕ ( X ) | ϕ
( X es un vector en tres dimensiones)

La acción de los momentos canónicos (en t = 0 )es dado por:

Π ^ ( X ) | ϕ = d d ϕ ( X ) | ϕ
Para un hamiltoniano cuadrático:
H ^ = d X Π ^ ( X ) 2 + d X d y Φ ^ ( X ) Ω ( X , y ) Φ ^ ( y )
( Ω puede verse como una matriz de masa de dimensión infinita)

El funcional de vacío tiene la forma:

Ψ Ω ( ϕ ) = ϕ | Ω = d mi t 1 4 ( Ω π ) mi 1 2 d X d y Φ ( X ) Ω ( X , y ) Φ ( y )
Esta es una generalización de dimensión infinita de la función de onda de vacío del oscilador armónico.

En infinitas dimensiones, un funcional de onda con diferente matriz de masa será ortogonal al funcional de onda de vacío ya todo su estado excitado, por lo que pertenecerá a otro sector de superselección. Hay ventajas en mantener una matriz de masa general porque en infinitas dimensiones permiten un tratamiento aproximado de campos que interactúan débilmente a través de una transformación de Bogoliubov. Sin embargo, por simplicidad, continuaré con una matriz de masa diagonal:

Ω ( X , y ) d ( X y )

(El vacío correspondiente y su funcional de onda se denotarán por: | 0 y Ψ 0 ( ϕ ) respectivamente). Además, no me molestaré con las infinitas normalizaciones.

En analogía, los operadores de creación y aniquilación de partículas están dados por:

A ( X ) = 1 2 ( Φ ^ ( X ) + i Π ^ ( X ) )
A ( X ) = 1 2 ( Φ ^ ( X ) i Π ^ ( X ) )

Por lo tanto, la amplitud requerida viene dada por:

ϕ | X = ϕ | A ( X ) | 0 >= ϕ ( X ) ϕ | 0 = ϕ ( X ) mi 1 2 d X ϕ ( X ) 2

La representación de Schrödinger de un estado coherente viene dada por analogía con el oscilador armónico:

Ψ α ( ϕ ) = norte mi 1 2 d X ( Φ ( X ) 2 α ( X ) ) 2
y el estado coherente viene dado por:

| α = D ϕ Ψ α ( ϕ ) | ϕ

(La integración es sobre todas las configuraciones de campo). Es fácil ver eso | α es un estado propio del operador de aniquilación.

Así obtenemos:

α | X = D ϕ Ψ α ( ϕ ) ϕ | X = α ( X )

En sistemas cuánticos descritos por estados coherentes de teorías de campo, la función α ( X ) suele denominarse función de onda macroscópica. La amplitud requerida tiene una expresión muy simple en términos de la función de onda macroscópica.

Interpretación de los resultados

La amplitud solicitada ϕ | X en la pregunta hay una superposición entre una configuración de campo ϕ y un solo estado de partícula. La superposición calculada es proporcional a ϕ ( X ) mi 1 2 d X ϕ ( X ) 2 . Este resultado, también calculado por Jackiw, tiene sentido, ya que es proporcional a la intensidad del campo en X . Sin embargo, esta amplitud será en general pequeña a menos que la configuración ϕ está en su punto máximo X . Por lo tanto, en la práctica, esta amplitud debería ser útil solo cuando el sistema está en un estado agrupado alrededor de X , por ejemplo en un estado solitónico. (Si me hubiera ocupado adecuadamente de las normalizaciones, la integral espacial, el módulo cuadrado del resultado, se habría normalizado automáticamente a 1 ).

El último cálculo estaba destinado a encontrar la amplitud α | X , en un estado coherente más bien en un estado de Schrödinger. La penúltima ecuación es solo el cambio de base entre la base de estado coherente y la base de Schrödinger. El campo complejo α se denomina función de onda macroscópica por la siguiente razón:

En un sistema no relativista descrito por un campo complejo de Schrödinger, el momento canónico es igual al conjugado complejo del campo: Π ^ ( X ) = Φ ^ ( X ) . Ya que debemos tener [ Φ ^ ( X ) , Π ^ ( X ) ] = d ( X X ) , debemos interpretar el campo como el operador de aniquilación y su cantidad de movimiento como el operador de creación (no como en el caso relativista, donde son combinaciones de operadores de creación y de aniquilación). En este caso, el valor esperado del campo en la base de estado coherente será:

α | Φ ^ ( X ) | α = α | A ( X ) | α = α ( X )
(La identidad A ( X ) | α = α ( X ) | α , válido en la base de estado coherente, fue utilizado).

En sistemas relativistas como la radiación o los plasmas relativistas todavía podemos llamar a la función α ( X ) una función de onda macroscópica aunque el razonamiento anterior no es válido.

La razón por la que les di también el resultado en una base de estado coherente es que cada vez que un sistema distribuido está en un estado coherente (como un condensado de Bose-Einstein); la función de onda macroscópica es una cantidad medible.

Muchas gracias por la completa respuesta y perdón por mi respuesta lenta, me tomé un tiempo para pensarlo. Mi pregunta principal es: ¿puedo confiar en que esta forma de generar estados propios de posición dará los resultados deseados? Dado que (hasta donde yo sé) las densidades de probabilidad no se han probado para partículas relativistas, básicamente esta pregunta es equivalente a "¿se reduce a la expresión QM en el límite no relativista?" Conecté la expresión para un campo escalar real libre. Obtuve <x|x'>=0 siX X, t =t . Sin embargo, los autoestados x dependen del tiempo. ¿Cómo puedo entender esto?
La descripción dada en la respuesta es completamente estática. Las cantidades en el texto se refieren at = 0 como se indica en el artículo de Jackiw. Por supuesto, tienes el hamiltoniano y puedes adelantar a los operadores en el tiempo. La elección que hice paraΩ no es relativista. Para tener un modelo relativistaΩ debe tomarse como2+metro2 . Por una expresión simple quise decir que la amplitud (no la probabilidad) que se requería en la pregunta es exactamente la función de onda macroscópica en la base de estado coherente (en la base de configuración de campo hay un factor gaussiano adicional).
continuación Los efectos de una función de onda macroscópica son observables en sistemas con coherencia cuántica de largo alcance. En mi opinión, estos efectos proporcionan una medida indirecta de las amplitudes que tienes en mente, ya que en sistemas con una gran cantidad de partículas, es difícil controlar una sola partícula; sin embargo, es más fácil controlar algún comportamiento colectivo.
No creo que esto sea correcto. Por ser una densidad de probabilidad,| <ψ | X>|2 debería integrarse a 1, pero su expresión no.
@Arnold Neumaier tiene constantes de normalización en las expresiones de los funcionales de onda que controlan la normalización de la función de onda en la última expresión, que debería ser una amplitud de probabilidad. No me molesté con las normalizaciones (algunas de ellas solo se pueden definir como límites) tampoco Jackiw, pero se pueden cuidar perfectamente desde el principio.
No entendí el papel de la función de onda macroscópica, ya que definimos los estados propios de posición, asumí| ϕ | x|2 era la probabilidad. Si ese es el caso, entonces ¿dóndeα ( x ) ¿Adelante? También me interesaría saber si se puede demostrar que este método se limita a la predicción de QM en el caso no relativista. Siento molestarte con tantas preguntas.
He preparado una respuesta a este comentario como una actualización en el texto principal.
Gracias, solo he leído un poco sobre lagrangianos que pordS= 0 proporcione la ecuación de Schrödinger (creo que eso es lo que quiso decir con un campo de Schrödinger complejo?) y pronto duplicaré sus pasos en casa. Ahora entiendo la razón por la que mencionaste los estados coherentes. Como para| x | ϕ|2 Reduciendo a la expresión no relativista, ¿estaba esto implícito en su respuesta al mencionar el campo de Schrödinger?
Para que quede claro, la razón por la que estoy tan interesado en el límite no relativista es que, por lo que puedo decir, este no es un enfoque estándar para encontrarρ ( x ) , principalmente porque parece que no hay un enfoque estándar: obtengo respuestas diferentes de cada profesor. Dado eso, parece importante verificar su consistencia antes de agregarlo con confianza a mi cerebro.
Mencioné el caso no relativista en el último párrafo de la respuesta para motivar el término: "Función de onda macroscópica", generalmente utilizada en superconductividad, donde las personas usan la teoría cuántica de campo no relativista: (En una teoría galileana no relativista, es el valor esperado del operador de campo en un estado coherente). Sin embargo, no creo que haya ninguna diferencia en la interpretación de la amplitud.ϕ | x entre los casos relativistas y no relativistas.
Hola. Con un poco más de experiencia en QFT, vuelvo a esta pregunta para finalmente comprender completamente la respuesta. Aunque podría estar perdiéndome algo, veo un problema con las propiedades de transformación de esta expresión. cuando en un estado| ψ> , por esta prescripción la densidad de probabilidad en x es| <ψ | X>|2= | < ψ | ( ϕ ( X ) - yo Π ( X ) ) | 0 >|2 . Como los campos se transforman como escalares, esta expresión debería transformarse como un escalar. Sin embargo, la densidad de probabilidad debería transformarse como el componente 0 de un 4vector. Esto sugiere que la expresión podría no ser correcta en general.
El formalismo canónico no es manifiestamente invariante de Lorentz, por ejemplo, en la definición de los momentos canónicosΠ =dLdΦ˙ , la derivada funcional es con respecto a una derivada temporal del campo. En consecuencia, las relaciones canónicas de conmutación entre los operadores de aniquilación y creación tampoco son manifiestamente relativistas:[ un (X⃗ ) ,A(y⃗ ) ] =d3(X⃗ y⃗ ) .
continuación Por lo tanto, la densidad de probabilidad debería ser un cuarto componente de un4 - vectores. Ahora, cómo ver eso: la invariancia de Lorentz no se pierde, solo no se manifiesta covariantemente. Primero, uno puede trabajar con el formalismo Crnković-Witten del espacio de fase covariante. O alternativamente, muestre explícitamente, la invariancia de Lorentz no manifiesta. Para este propósito, necesitamos construir la corriente de probabilidad correspondiente a la densidad de probabilidad anterior y construir las transformaciones de Lorentz explícitas.
¡Gracias por la respuesta! Por lo que entiendo, usted está diciendo que| ψ | x|2 en este formalismo se transforma como un componente de 4 vectores, pero eso no es obvio a partir de la forma (no manifiesto). Sin embargo, en mi comentario del 8 de diciembre, proporcioné pruebas de que no se transforma como un vector de 4. Esto significaría que su argumento no funciona. ¿Hubo una falla en mi razonamiento en ese comentario?
Como les escribí, la afirmación de queΠ ( x ) es un escalar es incorrecta, porque es la derivada de un escalar (el Lagrangiano), con respecto a una derivada en el tiempo .
Gracias por la ayuda. Estás bien,Π no es un escalar. Pero la expresión sugerida todavía tiene dos problemas que veo: 1. todavía no se transforma correctamente, como1t=1γ1t daΠ(X) = γΠ (X) , entonces la nueva amplitud esρ(X) = | ψ (X) | ( ϕ (X) yo γΠ (X) ) | 0 |2γρ (X) , dóndeρ(X) = γρ (X) es la ley de transformación correcta bajo un impulso. 2. el momento canónico no está definido de manera única, por ej. sumando una derivada total aL lo cambia
Por favor, note primero que el momento canónico es la derivada del Lagrangiano con respecto a la cuarta componente del gradiente del campo; no tiene que ser un cuarto componente de un vector por sí mismo. Por ejemplo, para un campo spinor, el momento canónico es el campo conjugado. Para un campo escalar, el momento canónico es de hecho un cuarto componente de un vector:π=ϕ˙ , es la cuarta componente del vectormϕ ; pero luego, bajo una transformación general de Lorentz, se mezcla con los componentes espaciales demϕ .
De todos modos, esta cantidad no parece transformarse correctamente, ¿o me estoy perdiendo algo?
Otra cosa a considerar: la relación de conmutación por sí sola no puede justificar la interpretación de partículas. El parun ( x ) yA( X ) todavía satisfaría la misma relación de conmutación en elϕ4 modelo con interacción arbitrariamente fuerte, donde ciertamente no son1 -operadores de creación/aniquilación de partículas. Incluso en el campo escalar libre (relativista), el operadorun ( x ) no aniquila el estado de vacío. Ambas cosasuna ( x ) | 0 yA( X ) | 0 son estados de una sola partícula en ese modelo, pero no están localizados en un punto (aunque los operadores sí).